Das "Photon"-Paradoxon oder wie die Richtung eines Photons umkehrt

Ich

Registriertes Mitglied
Ein Boost ist keine Beschleunigung, sondern eine Koordinatentransformation in ein bewegtes Bezugsytem; da wird nichts real beschleunigt.
Ja, aber wenn es um Dinge wie Thomas-Präzession geht, dann entspricht der Bezugssystemwechsel einer tatsächlichen Beschleunigung.
Da kommt die Tag noch mal was von mir, nur vorab mal gefragt, wenn ich einen Vektor habe auf einer Kugeloberfläche und denn dann vom Äquator über einen Pol wieder zum Äquator verschiebe und dann dem Äquator entlang zum Startplatz, dann hab ich den auch um 180 Grad gedreht. Nur durch das Verschieben. Ich suche da nach Bildern, Analogien, passt das wo? Hat diese Rotation was mit der Vierdimensionalität der Raumzeit zu tun?
Lorentztrafos allgemein sind Drehungen in der Raumzeit. Boosts sind Drehungen in den xt, yt, zt - Ebenen, also mit einer zeitartigen Richtung. Drehungen in einer Ebene mit zwei raumartigen Richtungen sind normale Drehungen.
Und wie bei allen Drehungen gilt auch hier, dass die Reihenfolge zählt, wenn ich mehrere hintereinander ausführe.
Stelle dir einen Würfel auf der 6 vor, den du erst nach rechts und dann nach vorne um 90° drehst. Er liegt dann z.B. auf der 3.
Wenn du stattdessen erst vor und dann nach rechts drehst, liegt er an derselben Stelle, aber auf der 2. Also verdreht gegenüber dem ersten Fall.
Genau das ist der "Mechanismus", der hier am Werk ist.
 

mojorisin

Registriertes Mitglied
Moin Mojorisin,

schön dass es nun geklappt hat, und auch ein Moin an alle anderen hier.


Also so wie ich es verstehe ist ein Lorentz-Boost eine Beschleunigung. Und es gibt auch die "normale" Lorentztransformation zwischen zwei zueinander bewegten Systemen.

Gruß in die Runde.

Ein Lorentzboost ist eine bestimmte Art der Lorentztransformation und keine Beschleunigung. Die Gruppe Der Lorentztransformationen beinhaltet räumliche Drehung und Spiegelung sowie die Transformation zwischen zueinander bewegten Inertialsystemen. Das letztere sind die sogenanten speziellen Lorentztransformationen oder Lorentzboosts (oftmals aber allgemein als Lorentztransformation bezeichnet.)

https://de.wikipedia.org/wiki/Spezielle_Lorentz-Transformation

Man kann mehrer Lorentztransformationen miteinander verknüpfen, dabei kommutieren diese aber nicht, das heißt:
Zuerst ein Boost und dann ein Drehung ist nicht dasselbe wie wenn ich Drehe und dann eine Boost durchführe.
Auf der anderen Seite kann ich aber reine Boosts aneinanderreihen die dann dasselbe bewirken wie ein Boost und eine Drehung.

Da Beschleunigungen sehr kompliziert zu handeln sind bin ich in meinen vorigen Posts von "aufgereihten" fest installierten Beobachtern ausgegangen die in das jeweilige "Nachbarsystem mit einem einzigen" Boost transformieren können. Vom Ersten zum Letzten kommt man dabei durch Aneinanderreihungh von reinen Lorentzboosts die aber nicht alle parallel sind. Vergleicht mann dann das Koordinantesystem vom Ersten und Letzten erkennt man das diese räumlich gegeneinander verdreht sind.

Das ist im krassen Gegensatz zur Galileitransformation, bei der reine (nichtparallele) Geschwindigkeitstransformationen nicht dazu führen das sich die Koordinatensysteme gegeneinder verdrehen.
 
Zuletzt bearbeitet:

mojorisin

Registriertes Mitglied
@Ich

...weil Galileotrafos Scherungen sind und keine Drehungen.

Ich habe mich etwas ungenau ausgedrückt. Auch die Galileitransforamtion enthalten natürlich Drehungen und Spiegelungen aber auch die Translationen. Was ich sagen wollte ist das eine Kombinatione nichtparalleler Translationen aber keine Drehung verursachen.

Was mir aber nicht bewusst ist das Galileitransformationen Scherungen sind. Hast du da mehr Info?
 

TomS

Registriertes Mitglied
Auch die Galileitransforamtion enthalten natürlich Drehungen und Spiegelungen aber auch die Translationen. Was ich sagen wollte ist das eine Kombinatione nichtparalleler Translationen aber keine Drehung verursachen.
Es geht auch nicht um nichtparallele Translationen sondern um nicht-parallele Boosts. Translationen sind sowieso keine Elemente der Lorentz- sondern der Poincaregruppe.

Auf der anderen Seite kann ich aber reine Boosts aneinanderreihen ...
Genau, es geht um Boosts.

EDIT: Die Hinterianderausführung mehrerer Galilei-Boosts liefert wieder einen Boost; diese bilden eine Abelsche Untergruppe:

$$ r \to r_1 = r + v_1 t \to r_2 = r_1 + v_2 t = r + (v_1 + v_2) t $$
 
Zuletzt bearbeitet:

mojorisin

Registriertes Mitglied
Es geht auch nicht um nichtparallele Translationen sondern um nicht-parallele Boosts. Translationen sind sowieso keine Elemente der Lorentz- sondern der Poincaregruppe.

Das stimmt natürlich. Translationen haben da nichts verloren.

EDIT: Die Hinterianderausführung mehrerer Galilei-Boosts liefert wieder einen Boost; diese bilden eine Abelsche Untergruppe:
Ich habe das Wort Galileo-Boost noch nie gehört, aber stimmt natürlich. ALso der Unterschied von Lorentz- zu Galilei-boost ist auf jeden Fall die Kommutativität, sprich die Gruppe der Lorentzboosts ist nichtabelsch. Kann man das allgemein stehen lassen?

Aber das mit den Scherungen braucht noch noch etwas Zeit bei mir. Ich komme aus eher aus den Materialwissenschaften und Scherungen haben für mich immer was mit Deformation zu tun.
Galilei-Transforamtionen als Scherung zu betrachten, dazu muss ich mich tiefer einlesen :)
 

TomS

Registriertes Mitglied
Ich habe das Wort Galileo-Boost noch nie gehört, aber stimmt natürlich. Also der Unterschied von Lorentz- zu Galilei-boost ist auf jeden Fall die Kommutativität, sprich die Gruppe der Lorentzboosts ist nichtabelsch. Kann man das allgemein stehen lassen?
Die Lorentzboosts bilden keine Gruppe. Wenn dem so wäre, dann müsste das Produkt zweier Lorentzboosts ja wieder ein reiner Lorentzboost sein. Wie du oben gesehen hast, enthält das Produkt zweier Lorentzboosts jedoch auch eine Drehung.

Es ist übrigens einfacher, sich das mittels der Algebra bzw. der Generatoren klar zu machen.
 

mojorisin

Registriertes Mitglied
Die Lorentzboosts bilden keine Gruppe. Wenn dem so wäre, dann müsste das Produkt zweier Lorentzboosts ja wieder ein reiner Lorentzboost sein. Wie du oben gesehen hast, enthält das Produkt zweier Lorentzboosts jedoch auch eine Drehung.

Es ist übrigens einfacher, sich das mittels der Algebra bzw. der Generatoren klar zu machen.

Ich muss mir mehr Zeit nehmen meine Post auszuarbeiten, sonst kommt so Zeugs dabei raus. Danke fürs Richtigstellen.
 
Oben